last day (21 days later) » 

10:02 PM
Okay
So I was thinking about these chiral currents they give
and how those expressions relate to zilch
 
Okay
So ACM
We've got some expression of some conserved current or whatever
and to me, it looks like it's of the form $(\mathcal L_\xi K^\mu_\nu)\xi^\nu$
I didn't check this yet but okay, let's assume it is
Now my question is whether this expression is somehow natural or to be expected to yield a conserved current
Also @EmilioPisanty so expression 2.14 in the Anco paper is of the form $A^\mu_\nu \xi^\nu$ for some tensor $A^\mu_\nu$ right
 
as I said to ACM just now, I think $A$ is the Lie derivative of some tensor $K^\mu_\nu=*F^{\mu\sigma}F_{\nu\sigma}-F^{\mu\sigma}*F_{\nu\sigma}$
 
Weird, I can't reply to messages here, either, but can in the h bar
 
10:07 PM
Very strange... could it have something to do with HSM? (this chat room is a hsm room because I'm a mod there and I made this room)
 
Yeah, I can't reply either
 
Crappy... :P
One of you could make a room?
 
I don't see how that expression would be natural, however, $L_\xi$ is indeed the change of $K$ under the flow induced by $\xi$.
 
Yeah I know
 
So if $\xi$ is some symmetry, I'd expect $L_\xi K$ to vanish for the associated conserved quantity, I think
 
10:09 PM
the first thing I wonder is... since $\xi$ is a symmetry of my theory and the tensor $K$ (I just gave the expression, $F$ are the standard field strengths for EM) should therefore be invariant, don't we have $\mathcal L_\xi K$ is zero?
 
^lol right, that's what I thought
 
@Danu OK, reload the page and it'll work
 
@Danu Yeah!
 
Since we're here already, shall we keep on this one then?
 
10:12 PM
Err sure?
So, there are two issues for now
1. What is $K$ in terms of $Z$? It seems like $Z$ is almost the derivative of $K$
(maybe it is, I didn't check)
2. What is this current, and how do we get it from $K$?
Also @ACuriousMind $Z$ is given by $Z^\mu_{\nu\rho}=*F^{\mu\sigma}F_{\sigma\nu,\rho} - F^{\mu\sigma}*F_{\sigma\nu,\rho}$
 
Hang on, what's $K$ here?
 
I defined it above
the same thing as $Z$ but without the derivatives (so one less index)
 
oh, yeah, I see it
hang on, I need paper to test stuff out
 
Yeah me too
I'm taking the derivative and the Lie derivative of $K$
 
@Danu If $K$ is not the conserved quantity associated to $\xi$, then the derivative isn't expected to vanish - e.g. four-momentum is conserved, but it changes its value under boosts (because it is the conserved quantity of translations, not boosts)
 
10:20 PM
Right, right. So that solves one thingy
Assuming the expression is actually the Lie derivative of $K$ this does give something nontrivial
Now, why do we care about the Lie derivative of $K$ under this trafo?
Also ACM are you interested in having the (most) relevant links to papers?
There's one by Kibble (from the 60s) and one newer one (more mathematical)
 
@Danu Are there more than the ones in the question?
 
@ACuriousMind Not really
There's a few more but I think the question's got all the relevant ones
 
@ACuriousMind The question has more than what you need
the Kibble and Anco papers are the only ones I found useful so far.
So...
$$ 2\partial_\rho K^\mu_\nu=\big[\epsilon^{\mu\sigma\alpha\beta}F_{\nu\sigma}-\epsilon_{\nu\sigma\alpha\beta}F^{\mu\sigma}\big]\partial_\rho F_{\alpha\beta}+\big[\epsilon^{\mu\sigma\alpha\beta}\partial_\rho F_{\nu\sigma}-\epsilon_{\nu\sigma\alpha\beta}\partial_\rho F^{\mu\sigma}\big] F_{\alpha\beta}$$
Any cancellations, anyone?
 
No, don't unpack the $\ast F$
 
Why not; I thought it'd help me see cancellations faster?
"packing it back up" is simple anyways
 
10:29 PM
You can get $\partial_\rho K^\mu_{\ \nu}$ in terms of $Z$ and a transpose
$\partial_\rho K^\mu_{\ \ \nu}=Z^\mu_{\ \ \nu\rho}-Z_{\nu\ \ \rho}^{\ \ \mu}$
 
Also whoops I typo'd on the definition of $Z$ earlier ACM, watch out
@EmilioPisanty Right
 
There might be other symmetries to make this easier
Yeah, the two terms should cancel
 
also
I think we should work with -1 times the equation 2.14 in Anco
because Kibble defines his contraction of the $F$'s the other way around
so contracted index first on the second factor i.e. $F^{\mu\sigma}*F_{\sigma\nu}$ etc
you okay with that convention?
 
Yeah, that's a good point
 
So let's stick to these definitions:
$$Z^\mu_{\nu\rho}:= *F^{\mu\sigma}F_{\sigma\nu,\rho}-F^{\mu\sigma}*F_{\sigma\nu,\rho}$$
 
10:37 PM
Yes
 
and $$K^\mu_\nu:=*F^{\mu\sigma}F_{\sigma\nu}- F^{\mu\sigma}*F_{\sigma\nu}$$
 
Yeah, that's what I'm working with
good spot on the sign difference w.r.t. Anco
 
I also have verified now that $$\partial_\rho K^\mu_\nu=Z^\mu_{\ \ \nu\rho}-Z_{\nu\ \ \ \rho}^{\ \ \mu}$$
now, what did you mean when you said something should cancel?
 
Not sure anymore :|
Kibble reckons that $Z^{\mu\nu\rho}=Z^{\nu\mu\rho}$, eq. 6
 
Yes, that follows from his identity (2)
hmm yes, I see... that'd be not-so-nice huh
So the derivative of $K$ should actually vanish
 
10:49 PM
Yeah, that's pretty strange. So $K$ should be constant in space.
 
No, not necessarily constant in space (unless you already know it's constant in time)
Is your $E\cdot B$ constant in time?
It's conserved but not necessarily constant
 
Yeah, but all the partial derivatives of $K$ vanish. I'm afraid it's just zero by definition or something.
 
Oh, I'm retarded, sorry
Conserved is when you contract the indices lol
So you're right, $K$ is somehow trivial
 
$Z_{000}$ is something like $\mathbf E\cdot\dot{\mathbf B}-\mathbf B\cdot\dot{\mathbf E}$, and you've taken out the derivatives
 
you can't concatenate styles for letters without braces
@EmilioPisanty Right, right.
Okay, so then it's back to scratch; find some way to relate 2.14 to what we know
I still want to write it as ($\xi$ times) the Lie derivative of something
 
10:58 PM
@Danu Yeah, I'm more worried in terms of how does 2.14 emerge from Noether's theorem
 
Yeah, maybe it's not directly through a Lie derivative after all
In the paper you linked in the other room, we do find the following:
> The dual symmetry is associated with optical helicity so the connection between special case of zilches, optical chirality, with helicity becomes more clear
So I guess we have to perform an infinitesimal duality transformation
 
Yeah, there's definitely going to be a duality symmetry involved
 
and then use the usual Noether's theorem method of deriving the current
 
But I think you also need to couple it with some conformal transformation
 
Right, because how else are we going to get that $\xi$ in there
 
11:03 PM
So maybe the zilch is the Noether current for duality plus a Lorentz transform?
 
you mean a conformal transformation
what do you think @ACuriousMind?
 
Yeah, in the general case, but I suspect that zilch as in Lipkin and Kibble only needs the special case of Lorentz transformations
so $\xi^\mu=a^{\mu\nu}x_\nu$.
It would have the huge advantage of being relatively easy to implement, so one can just check
 
What is your reason for believing that?
Now, what does an infinitesimal duality trafo look like? :P
I've got this nice-n-general formula for the Noether current associated to a combined field-and-coordinate transformation
 
@Danu Mostly the fact that you need to get rotations involved somehow
So you can tap into the pseudoscalarness of the thing
 
If $\delta \mathcal L=\partial_\mu K^\mu$ for some $K^\mu$ that vanishes "at infinity" then the general current is:
$$ J^\mu:=\frac{\partial \mathcal L}{\partial \partial_\mu\phi^i} \delta\phi^i - \frac{\partial \mathcal L}{\partial (\partial_\mu \phi^i)}\partial_\nu \phi^i\delta x^\nu +\mathcal L\delta x^\mu -K^\mu $$
So... what is the infinitesimal duality transformation like? :P
 
11:13 PM
So in terms of the vector fields, it's $\mathbf E'=\cos\theta \mathbf E-\sin\theta \mathbf B$, $\mathbf B=\sin\theta \mathbf E+\cos\theta \mathbf B$
 
In terms of the vector potential, by any chance?
 
So I imagine something like $F'^{\mu\nu}=\cos\theta F^{\mu\nu}\pm \sin\theta {}^\ast F^{\mu\mu}$
no, it's on the force fields
At $\theta=\pi/2$ you need to completely swap them
 
@Danu Yeah, I saw that one late last night, it's on the queue
 
@EmilioPisanty I guess in terms of either $F$ or $A$ is fine
 
11:16 PM
@Danu Yeah, that one is just a guess, though
Or actually
should work
 
That's a funky link
 
That looks as if the duality transformation really "wants" to be a $\mathrm{SO}(2)$ transformation acting on $F \choose \ast F$ as rotation by $\theta$
 
It is, sorry; I'm logged into my uni stuff
@ACuriousMind Yeah, it does want to be that
 
@ACuriousMind Yeah, exactly
 
11:22 PM
So the infinitesimal version is just $F + \theta (\ast F)$ for small $\theta$.
 
Bizarre but that's what it says, I guess
@ACuriousMind Yeah
 
In fact, however, Maxwell's equations are invariant under $GL(2)$ transformations on the $F, *F$ vector
not just under rotations
 
Yeah, that's true. So does the bigger group get you anything?
 
@ACuriousMind Should be minus, no?
(depends on the direction of the rotation, I guess; I'm taking counterclockwise to be standard)
 
The scaling has to give you fairly trivial stuff
 
11:24 PM
@Danu There's room for a couple of sign conventions here ;)
 
@Danu counterclockwise is an odd term for $\begin{pmatrix}F\\\ast F\end{pmatrix}$ =P
 
@EmilioPisanty The rotations (first entry of vector $=: x$, duhhhh ;) )
So let's just stupidly apply this:
$$ F\mapsto F-\theta *F_{\mu\nu}$$ and then the Lorentz transformation (forgot infinitesimal version, sigh... will re-derive again)
oh yeah lol it's the antisymmetric thing
 
@Danu ??
 
@EmilioPisanty Infinitesimal Lorentz transformation thingy
@EmilioPisanty The paper by Barnett et al. introduces an auxiliary field btw, which I don't like.
 
Oh, yeah. $x^\mu\mapsto x^\mu+\theta a^{\mu\nu}x_\nu$, $a^{\nu\mu}=-a^{\mu\nu}$.
@Danu The $\mathbf C$ thing?
That feels pretty inevitable if you want to do the duality transformation on the potentials
whether it's actually necessary is a good question though
 
11:39 PM
So erm now we should just check out da Noether current right; the thing is that this is not so nice since we don't have the transformation in terms of $A_\mu$
Am I being stupid here?
 
You can always put it in directly
$\ast F$ is an explicit function of $A$
 
Yeah, but our transformation is in terms of $F$ and it doesn't directly (or even uniquely?) define one for $A$
 
Oh, as in, you want $A\mapsto A'=\cdots$?
 
mhm
 
oooh, good question
 
11:45 PM
I don't understand how to get my Noether current otherwise :P
If you look at my expression for $J^\mu$ you'll see that it involves a bunch of derivatives w.r.t. the derivatives of fields; there are clearly no derivatives of $F$
 
Oh, so the $\phi^i$ are just the $A^\mu$?
got it
 
The $\phi^i$ are the "fundamental fields" appearing in $\mathcal L$
 
ok, yeah
 
(they'll always have a kinetic term, hence a derivative somewhere)
 

  last day (21 days later) »